Question

In: Economics

QUESTION 1 Two firms compete by choosing price. Their demand functions are Q1 = 20 -...

QUESTION 1

Two firms compete by choosing price. Their demand functions are Q1 = 20 - P1 +P2 and Q2 = 20 - P2 +P1 where P1 and P2 are the prices charged by each firm, respectively, and Q1 and Q2 are the resulting demands. Note that the demand for each good depends only on the difference in prices; if the two firms colluded and set the same price, they could make that price as high as they wanted, and earn infinite profits. Marginal costs are zero. Suppose the two firms set their prices at the same time. Find the resulting equilibrium. What price will each firm charge, and what will its profit be?

P1=
P2=
Profit1=
Profit2=

0.5 points   

QUESTION 2

Two firms compete by choosing price. Their demand functions are Q1 = 112 - 4P1 + P2 and Q2 = 112 - 4P2 + P1 where P1 and P2 are the prices charged by each firm, respectively, and Q1 and Q2 are the resulting demands. Each firm has a marginal constant produce at a zero constant average and marginal cost (AC = MC = $0). Suppose the two firms set their prices at the same time. What price will each firm charge, how much will it sell, and what will its profit be? (Hint: Maximize the profit of each firm with respect to its price.)

P1=

P2=

Profit1=

Profit2 =

0.5 points=   

Solutions

Expert Solution

Firm 1:

Q1 = 20 - P1 +P2

MC = 0

Total Revenue TR1 = P1*Q1 = P1(20-P1+P2)

Profit = P1(20-P1+P2)

= 20P1 - P12 + P1P2

d/dP1 = 20 - 2P1 + P2

Set d/dP1 = 0 we get,

2P1 = 20 + P2

P1 = 10 + 0.5P2 ..............1

Firm 2

Q1 = 20 - P2 +P1

MC = 0

Total Revenue TR2 = P2*Q2 = P2(20-P2+P1)

Profit = P2(20-P2+P1)

= 20P2 - P22 + P1P2

d/dP2 = 20 - 2P2 + P1

Set d/dP2 = 0 we get,

2P2 = 20 + P1

P2 = 10 + 0.5P1

Put the value of P1 from equation 1 we get,

P2 = 10 + 0.5(10 + 0.5P2)

P2 = 10 + 5 + 0.25P2

0.75P2 = 15

P2* = 20

P1 = 10 + 0.5P2

P1* = 20

Profit of firm 1: 20P1 - P12 + P1P2 = 20*20 - 202 + 20*20 = 400 = Profit of firm 2

Answer 2:

Firm 1:

Q1 = 112 - 4P1 +P2

MC = 0

Total Revenue TR1 = P1*Q1 = P1(112-4P1+P2)

Profit = P1(112-4P1+P2)

= 1120P1 - 4P12 + P1P2

d/dP1 = 112 - 8P1 + P2

Set d/dP1 = 0 we get,

8P1 = 112 + P2

P1 = 14 + 0.125P2 ..............1

Firm 2

Q2 = 112 - 4P2 +P1

MC = 0

Total Revenue TR2 = P2*Q2 = P2(112-4P2+P1)

Profit = P2(112-4P2+P1)

= 1120P2 - 4P22 + P1P2

d/dP2 = 112 - 8P2 + P1

Set d/dP2 = 0 we get,

8P2 = 112 + P1

P2 = 14 + 0.125P1

Put the value of P1 from equation 1

P2 = 14 + 0.125(14 + 0.125P2)

P2 = 14 + 1.75 + 0.015625P2

0.984375P2 = 15.75

P2 = 16

P1 = 14 + 0.125P2

P1 = 16

Profit of firm 1: 112P1 - 4P12 + P1P2 = 112*16 - 4*162 + 16*16 = 1792 - 1024 + 256 = 1024 = Profit of firm 2


Related Solutions

Two firms compete by choosing price. Their demand functions are Q_1=20-P_1+P_2 and Q_2=20+P_1-P_2 where P_1 and...
Two firms compete by choosing price. Their demand functions are Q_1=20-P_1+P_2 and Q_2=20+P_1-P_2 where P_1 and P_2 are the prices charged by each firm, respectively, and Q_1 and Q_2 are the resulting demands. Note that the demand for each good depends only on the difference in prices; if the two firms collude and set the same price, they could make that price as high as they wanted, and earn infinite profits. Marginal costs are zero. a. Suppose the two firms...
Consider two firms that sell differentiated products and compete by choosing prices. Their demand functions are...
Consider two firms that sell differentiated products and compete by choosing prices. Their demand functions are Q1 = 72 – 3P1 + 2P2 and Q2 = 72 – 3P2 + 2P1 where P1 and P2 are the prices charged by firm 1 and 2, respectively, and Q1 and Q2 are the corresponding demands. All production costs are assumed to be zero. (a) Suppose the two firms set their prices simultaneously and non-cooperatively. Find the resulting Bertrand-Nash equilibrium. What price does...
Two firms compete in a market by selling differentiated products. The demand equations are : q1...
Two firms compete in a market by selling differentiated products. The demand equations are : q1 = 75 – p1 +p2/2 q2 = 75 – p2 +p1/2 Assume that each firm has a marginal cost and average costs of 0 What market model do we use if each firm competes by simultaneously choosing price Are the two goods substitutes? . Solve for firm 1’s best response function. Solve for the equilibrium price and quantity. Would firm 1 still be able...
Two firms, Firm 1 and Firm 2, compete by simultaneously choosing prices. Both firms sell an...
Two firms, Firm 1 and Firm 2, compete by simultaneously choosing prices. Both firms sell an identical product for which each of 100 consumers has a maximum willingness to pay of $40. Each consumer will buy at most 1 unit, and will buy it from whichever firm charges the lowest price. If both firms set the same price, they share the market equally. Costs are given by ??(??)=16??ci(qi)=16qi. Because of government regulation, firms can only choose prices which are integer...
Question 4 Two firms compete as a Stackelberg duopoly. The demand they face is P =...
Question 4 Two firms compete as a Stackelberg duopoly. The demand they face is P = 40 − Q. The cost function for each firm is C(Q) = 4Q. What are the profits of the two firms? I believe the answer is πL = $162; πF = $81. however I need clear steps to understand how to understand the process.
1. Consider a market of homogeneous products in which firms compete on price. Demand in this...
1. Consider a market of homogeneous products in which firms compete on price. Demand in this market is given by q(p) = 50 -10p Consumers buy from the producer with the lowest price. If the prices of both firms are the same then they purchase from E. There are both an incumbent firm M and a potential entrant E which can produce the good at marginal costs 3 and 2 , respectively. Prior to entry, E must incur an entry...
Consider a price competition model with two firms, 1 and 2, whose demand functions are as...
Consider a price competition model with two firms, 1 and 2, whose demand functions are as follows: Q1 =48−3P1 +2P2 Q2 =80−4P2 +3P1 Each firm incurs costs; C1(Q1) = 8Q1 and C2(Q2) = 13Q2. a) Write down firms 1’s and 2’s profit functions. b) Compute and graph firms 1’s and 2’s best response functions as a function of the other firm’s prices. c) Find the Nash equilibrium of the game. d) Calculate the profit of each firm in the Nash...
If firms compete by choosing quantities of output, is there an advantage to moving later or...
If firms compete by choosing quantities of output, is there an advantage to moving later or sooner? How might a firm commit to go sooner?
1. Two firms compete in Cournot competition. Inverse demand in the market is given by P...
1. Two firms compete in Cournot competition. Inverse demand in the market is given by P = 1500 − 3 Q and each firm has constant marginal cost c = 420. a) Assuming there are no fixed costs, find the Cournot equilibrium market price and quantities produced by each firm. (20 points) b) Now suppose that each firm faces a non-sunk fixed cost of 20,000 if they produce at all. Would the firms still want to produce the amounts you...
Two firms, a and b, compete in a market to sell homogeneous products with inverse demand...
Two firms, a and b, compete in a market to sell homogeneous products with inverse demand function P = 400 – 2Q where Q = Qa + Qb. Firm a has the cost function Ca = 100 + 15Qa and firm b has the cost function Cb = 100 + 15Qb. Use this information to compare the output levels, price and profits in settings characterized by the following markets: Cournot Stackelberg Bertrand Collusion
ADVERTISEMENT
ADVERTISEMENT
ADVERTISEMENT